Download Which of the following patients would MOST likely present with

Survey
yes no Was this document useful for you?
   Thank you for your participation!

* Your assessment is very important for improving the workof artificial intelligence, which forms the content of this project

Document related concepts

Cardiac contractility modulation wikipedia , lookup

Electrocardiography wikipedia , lookup

Coronary artery disease wikipedia , lookup

Cardiac surgery wikipedia , lookup

Management of acute coronary syndrome wikipedia , lookup

Dextro-Transposition of the great arteries wikipedia , lookup

Quantium Medical Cardiac Output wikipedia , lookup

Transcript
Which of the following patients would MOST likely present with vague or unusual symptoms of
an acute myocardial infarction?!
!
A:!
55-year-old obese female!
B:!
66-year-old male with angina!
C:!
75-year-old male with hypertension!
D:!
72-year-old female with diabetes!
You selected A; The correct answer is D;!
!
Reason:!
!
Not all patients experiencing acute myocardial infarction (AMI) present with the classic signs
and symptoms one would expect. Middle-aged men often minimize their symptoms and attribute
their chest pain or discomfort to indigestion. Some patients, however, do not experience any
pain. In particular, elderly women with diabetes may present with vague, unusual, or atypical
symptoms of AMI; their only presenting complaint may be fatigue or syncope. Do not rule out a
cardiac problem just because a patient is not experiencing chest pain, pressure, or discomfort;
this is especially true in elderly females with diabetes.!
!
=====================================!
!
The energy setting for a biphasic AED:!
!
A:!
increases by 50 joules with each shock.!
B:!
is typically 360 joules.!
C:!
must be manually set by the EMT.!
D:!
is manufacturer specific.!
You selected A; The correct answer is D;!
!
Reason:!
!
A biphasic AED sends energy in two directions simultaneously. This is advantageous because it
produces a more efficient defibrillation than monophasic defibrillation, which sends energy in
only one direction, and uses a lower energy setting. The energy setting on a biphasic AED is
device specific, but typically ranges from 120 to 200 joules. AEDs are preprogrammed by the
manufacturer to deliver a specific amount of energy, which eliminates the need to manually set
the energy level. Some biphasic AEDs deliver the same amount of energy for each shock;
others may escalate the energy level for each shock. Refer to the manufacturer's
documentation that accompanies the AED you use regarding the amount of biphasic energy it
delivers.!
!
=====================================!
!
The automated external defibrillator (AED) should NOT be used in patients who:!
!
A:!
B:!
C:!
D:!
experienced a witnessed cardiac arrest.!
are apneic and have a weak carotid pulse.!
have a nitroglycerin patch applied to the skin.!
are between 1 and 8 years of age.!
You selected A; The correct answer is B;!
!
Reason:!
!
The AED is only applied to patients in cardiac arrest (eg, pulseless and apneic), whether the
arrest was witnessed or unwitnessed. According to the 2010 guidelines for CPR and Emergency
Cardiac Care (ECC), AEDs can safely be used in infants and children less than 8 years of age
in conjunction with a dose-attenuating system (energy reducer) and pediatric pads. However, if
pediatric pads and an energy reducer are unavailable, adult AED pads should be used. A
nitroglycerin patch is not a contraindication to the use of an AED; simply remove the patch (with
gloved hands) and apply the AED as usual.!
!
=====================================!
!
Which of the following would clearly be detrimental to a patient in cardiac arrest?!
!
A:!
Using a pocket face mask without high-flow oxygen!
B:!
Ventilating just until the chest rises!
C:!
Interrupting CPR for more than 10 seconds!
D:!
Performing CPR before defibrillation!
You selected A; The correct answer is C;!
!
Reason:!
!
Major emphasis is placed on minimizing interruptions in CPR. Even brief interruptions cause a
significant decrease in blood flow to the heart and brain. If you must interrupt CPR, do not
exceed 10 seconds. It is preferable to ventilate a patient with a pocket face mask attached to
high-flow oxygen, but failing to do so will not be nearly as detrimental as interrupting CPR for
extended periods of time. You should ventilate the patient just until the chest visibly rises;
ventilations that are too forceful or too fast can cause hyperinflation of the lungs, which may
reduce blood return to the heart. When caring for any patient in cardiac arrest, you should
immediately begin CPR, and then apply the AED as soon as possible.!
!
=====================================!
!
A patient who is experiencing an acute myocardial infarction:!
!
A:!
often complains of a different type of pain than a patient with angina.!
B:!
most often describes his or her chest pain as being sharp or tearing.!
C:!
has chest pain or discomfort that does not change with each breath.!
D:!
often experiences relief of his or her chest pain after taking nitroglycerin.!
You selected A; The correct answer is C;!
!
Reason:!
!
The type of chest pain or discomfort associated with acute myocardial infarction (AMI) is the
same that is experienced by patients with angina pectoris (eg, dull, crushing, pressure,
heaviness); thus, you cannot distinguish AMI from angina pectoris based solely on the type or
quality of pain. Furthermore, the pain associated with AMI, like that of angina, often radiates to
the arm, jaw, back, or epigastrium. Relative to other causes of chest pain or discomfort (eg,
pleurisy, pneumothorax), the pain associated with AMI and angina does not worsen or improve
when the patient takes a breath. Rest and nitroglycerin often relieve the pain associated with
stable angina, but are less likely to relieve the pain associated with AMI.!
!
=====================================!
!
When assessing a patient who complains of chest pain, which of the following questions would
you ask to assess the "R" in OPQRST?!
!
A:!
Did the pain begin suddenly or gradually?!
B:!
Is there anything that makes the pain worse?!
C:!
What were you doing when the pain began?!
D:!
Is the pain in one place or does it move around?!
You selected A; The correct answer is D;!
!
Reason:!
!
The “R” in OPQRST stands for radiation or referred pain. An appropriate way to determine
whether the pain radiates or not is to ask the patient if the pain remains in one place or if it
moves around. When determining if the patient has referred pain, ask him or her if he or she
hurts somewhere other than his or her chest. If you use the term “radiating pain,” chances are
the patient will not understand what you are asking.!
!
=====================================!
!
When an error occurs while using the AED, it is MOST often the result of:!
!
A:!
misinterpretation of the patient’s cardiac rhythm.!
B:!
excess patient movement during the analyze phase.!
C:!
malfunction of the microchip inside the AED.!
D:!
battery failure secondary to operator error.!
You selected A; The correct answer is D;!
!
Reason:!
!
AEDs have a high specificity for recognizing shockable rhythm (eg, V-Fib, pulseless V-Tach);
this means that they are highly reliable. It would be extremely rare for the AED to recommend a
shock when one is not indicated or fail to recommend a shock when one is indicated. When an
error does occur, it is usually the operator’s fault. The most common error is not having a
charged battery. To avoid this problem, many AEDs are equipped with an alarm that warns the
operator if the battery is not fully charged. Some of the older AEDs, however, are not equipped
with this feature. Therefore, it is important to check the AED daily, exercise the battery as often
as the manufacturer recommends, and always have a spare, fully-charged, battery on hand.!
!
=====================================!
!
When the vital organs of an 85-year-old patient need additional blood flow, the heart may not be
able to meet the increased need because:!
!
A:!
the reserve capacity of the heart is reduced.!
B:!
stroke volume increases as a person ages.!
C:!
the patient’s blood vessels become more elastic.!
D:!
blood pressure decreases as a person ages.!
You selected A; This is correct!!
!
Reason:!
!
Like other body systems, the cardiovascular system undergoes changes as we get older. As the
heart’s muscle mass and tone decrease, the amount of blood pumped out of the heart per beat
(stroke volume) is reduced. The residual (reserve) capacity of the heart is also reduced;
therefore, when the vital organs of the body need additional blood flow, the heart is less able to
meet that need as effectively as a younger person's heart. The vascular system also is affected
by the aging process. Arteriosclerosis (hardening of the arteries) can develop, affecting
perfusion of the tissues. In arteriosclerosis, the arteries become less elastic and more narrowed;
as a result, blood pressure typically increases with age.!
!
=====================================!
!
You are assessing a 70-year-old male who complains of pain in both of his legs. He is conscious
and alert, has a blood pressure of 160/90 mm Hg, a pulse rate of 110 beats/min, and
respirations of 14 breaths/min and unlabored. Further assessment reveals edema to both of his
feet and legs and jugular venous distention. This patient's primary problem is MOST likely:!
!
A:!
pulmonary edema.!
B:!
left heart failure.!
C:!
right heart failure.!
D:!
chronic hypertension.!
You selected A; The correct answer is C;!
!
Reason:!
!
If the right side of the heart is damaged, fluid collects in the body (edema), often showing in the
feet and legs. The collection of fluid in the part of the body that is closest to the ground is called
dependent edema. The swelling causes relatively few symptoms other than discomfort. Another
feature of right heart failure is jugular venous distention, which is an indication of blood backing
up into the systemic circulation. Left heart failure typically presents with shortness of breath due
to fluid in the lungs (pulmonary edema), which indicates blood backing up from the left side of
the heart into the lungs. In severe pulmonary edema, the patient may cough up pink, frothy
sputum. Right heart failure and/or left heart failure are also referred to as congestive heart
failure (CHF). Chronic hypertension cannot be established on the basis of a single blood
pressure reading.!
!
=====================================!
!
Which of the following statements regarding sudden cardiac arrest and ventricular fibrillation is
correct?!
!
A:!
High-quality CPR often reverses ventricular fibrillation if it is initiated within 2 minutes of
the onset.!
B:!
Most patients develop ventricular fibrillation within 10 minutes after the onset of sudden
cardiac arrest.!
C:!
Patients with ventricular fibrillation are typically unconscious, apneic, and have a weak
and irregular pulse.!
D:!
For each minute that defibrillation is delayed, the chance of survival decreases by as
much as 10%.!
You selected A; The correct answer is D;!
!
Reason:!
!
Ventricular fibrillation (V-Fib), a chaotic quivering of the heart muscle, is the most common
dysrhythmia that results in sudden cardiac arrest (SCA). In V-Fib, the heart is not beating
effectively and is not pumping blood; therefore, the patient will not have a pulse. The single most
important treatment for V-Fib is early defibrillation. Even if CPR is begun right at the time of the
patient’s collapse, the likelihood of survival decreases by as much as 10% for each minute that
defibrillation is delayed. CPR alone rarely, if ever, converts V-Fib to a cardiac rhythm with a
pulse. V-Fib is often a transient dysrhythmia, and the window of opportunity for successful
defibrillation is very narrow. After 10 minutes of cardiac arrest, most patients deteriorate to
asystole (absence of electrical and mechanical activity in the heart).!
!
=====================================!
!
Aspirin may be contraindicated in patients with:!
!
A:!
ibuprofen allergy.!
B:!
glaucoma.!
C:!
stomach ulcers.!
D:!
diabetes.!
You selected A; The correct answer is C;!
!
Reason:!
!
Aspirin (acetylsalicylic acid [ASA]) inhibits platelet aggregation, thus preventing clots from
forming or preventing an existing clot from getting bigger. Aspirin, in a dose of 160 to 325 mg,
should be administered to patients experiencing acute coronary syndrome (ie, unstable angina,
acute myocardial infarction) as soon as possible. Aspirin is absolutely contraindicated for
patients who are allergic to salicylates. Because aspirin prolongs bleeding time, it may be
contraindicated for patients with stomach ulcers; therefore, you should contact medical control
before giving aspirin to such patients. Aspirin is not contraindicated for patients with glaucoma
or diabetes. Ibuprofen, the active ingredient in Motrin and Advil, is a nonsteroidal antiinflammatory drug (NSAID), not a salycilate.!
!
=====================================!
!
Which of the following chambers of the heart has the thickest walls?!
!
A:!
Left atrium!
B:!
Right ventricle!
C:!
Right atrium!
D:!
Left ventricle!
You selected A; The correct answer is D;!
!
Reason:!
!
The left ventricle is the most powerful chamber of the heart. It does most of the work; therefore,
it has the thickest walls. A palpable pulse represents left ventricular contraction. Because the left
ventricle does most of the work for the heart, it demands and consumes a significant amount of
oxygen. This makes it the most common site for a heart attack (acute myocardial infarction).!
!
=====================================!
!
The chest pain associated with an acute coronary syndrome is often described as:!
!
A:!
cramping.!
B:!
sharp.!
C:!
pressure.!
D:!
stabbing.!
You selected A; The correct answer is C;!
!
Reason:!
!
The pain associated with acute coronary syndrome (ie, unstable angina, acute myocardial
infarction) is commonly described as a feeling of pressure or heaviness, discomfort, or as a
crushing pain. Stabbing or sharp (pleuritic) pain is often associated with conditions such as a
spontaneous pneumothorax or pulmonary embolism. Do not rule out a cardiac problem just
because the patient does not have the "classic" quality of pain, especially if the patient has a
history of cardiac problems.!
!
=====================================!
!
Which of the following statements regarding the automated external defibrillator (AED) is
correct?!
!
A:!
The AED should be applied to patients at risk for cardiac arrest!
B:!
AEDs will analyze the patient’s rhythm while CPR is in progress!
C:!
AEDs can safely be used in infants and children less than 8 years of age!
D:!
The AED should not be used in patients with an implanted defibrillator!
You selected A; The correct answer is C;!
!
Reason:!
!
According to the 2010 guidelines for CPR and Emergency Cardiac Care (ECC), the AED can
safely be used in infants and children less than 8 years of age. Although a manual defibrillator is
preferred in infants, an AED can be used. When using the AED in infants and children, you
should use pediatric pads and a dose-attenuating system (energy reducer); however, if these
features are not available, adult AED pads should be used. The AED should only be applied to
patients in cardiac arrest; if a patient is at risk for cardiac arrest, have the AED ready but not
applied. The AED will not analyze the cardiac rhythm if the patient is moving (ie, CPR is in
progress). AEDs can be used in patients with an automated implanted cardioverter/defibrillator
(AICD) or implanted pacemaker; ensure that the pads are at least 1” away from the implanted
device.!
!
=====================================!
!
Which of the following patients is the BEST candidate for the administration of nitroglycerin?!
!
A:!
A man with chest pain, expired nitroglycerin spray, and a blood pressure of 110/80 mm
Hg!
B:!
A woman who has taken three doses of prescribed nitroglycerin without relief of chest
pain!
C:!
A woman with chest pain, prescribed nitroglycerin, and a blood pressure of 104/76 mm
Hg!
D:!
An elderly man with crushing substernal chest pain and a blood pressure of 80/60 mm
Hg!
You selected A; The correct answer is C;!
!
Reason:!
!
Nitroglycerin should be administered to patients who have the prescribed, unexpired drug with
them and a systolic blood pressure of greater than 100 mm Hg. No more than three (3)
nitroglycerin tablets or sprays should be administered to a patient in the prehospital setting. An
expired medication should never be administered to any patient, even if the medication is
otherwise indicated for his or her condition.!
!
=====================================!
!
The position of comfort for a patient with nontraumatic chest pain MOST commonly is:!
!
A:!
semisitting.!
B:!
supine with the legs elevated slightly.!
C:!
lateral recumbent.!
D:!
on the side with the head elevated.!
You selected A; This is correct!!
!
Reason:!
!
As with most patients, the position of comfort for cardiac patients typically is the semisitting
(semi-Fowler) position. You should allow the patient to remain in the position of comfort both
during the assessment phase as well as throughout transport.!
!
=====================================!
!
When assessing a patient with a possible stroke, you should recall that:!
!
A:!
fibrinolytic therapy must be given within 6 hours following the stroke.!
B:!
right-sided weakness indicates a stroke in the right cerebral hemisphere.!
C:!
the patient may be unable to communicate, but can often understand.!
D:!
the majority of strokes involve bleeding into the brain tissue.!
You selected A; The correct answer is C;!
!
Reason:!
!
Some patients who have had a stroke may be unable to communicate (expressive aphasia), but
they can often understand what is being said around them; be aware of this possibility. Seventyfive percent of all strokes are caused by a blocked cerebral artery (ischemic stroke); strokes
caused by a ruptured cerebral artery (hemorrhagic stroke) are less common. Because the left
side of the brain controls the right side of the body and vice versa, right-sided weakness
(hemiparesis) indicates a stroke in the left cerebral hemisphere. Some patients who have had a
stroke may benefit from fibrinolytic (clot-buster) therapy; however, to be most effective, this
treatment must be given within the first 3 hours following the onset of the stroke.!
!
=====================================!
!
A 58-year-old man is found unresponsive by his wife. According to his wife, he was complaining
of a "dull ache" in his chest the day before, but refused to allow her to call 9-1-1. His blood
pressure is 70/50 mm Hg, his pulse is 120 beats/min and weak, and his respirations are 28
breaths/min and labored. Further assessment reveals that his skin is cool, pale, and clammy.
You should suspect:!
!
A:!
acute septic shock.!
B:!
hypovolemic shock.!
C:!
cardiogenic shock.!
D:!
pulmonary embolism.!
You selected A; The correct answer is C;!
!
Reason:!
!
The patient in this scenario likely experienced an acute myocardial infarction the day before.
However, because he refused medical care, the infarction has likely damaged a significant
portion of his heart, resulting in cardiogenic shock. Cardiogenic shock occurs when the heart
fails as a pump and can no longer meet the metabolic needs of the body; it has a very high
mortality rate. The patient's labored breathing is likely caused by pulmonary edema, which
occurs when blood backs up into the lungs because the heart cannot effectively pump. There is
no evidence of hypovolemia (ie, poor skin turgor) or sepsis (ie, fever). A pulmonary embolism
typically presents with an acute onset of pleuritic (sharp) chest pain and difficulty breathing, and
is also commonly associated with cyanosis. !
!
=====================================!
!
A middle-aged male was found unresponsive by his wife. When you arrive at the scene, you
assess the patient and determine that he is apneic and pulseless. You should:!
!
A:!
immediately apply the AED, analyze his cardiac rhythm, deliver a shock if indicated, and
begin CPR.!
B:!
perform CPR with a compression-to-ventilation ratio of 15:2, apply the AED, and request
backup.!
C:!
immediately begin CPR, reassess for a carotid pulse after 60 seconds, and then apply
the AED.!
D:!
begin CPR starting with chest compressions, apply the AED as soon as possible, and
request backup.!
You selected A; The correct answer is D;!
!
Reason:!
!
As soon as you determine that a patient is unresponsive, pulseless, and apneic, you should
begin CPR starting with chest compressions. The appropriate compression-to-ventilation ratio
for adult CPR (one- or two-rescuer) is 30:2. A compression-to-ventilation ratio of 15:2 is used for
two-rescuer infant and child CPR. Request a backup ambulance as soon as possible; however,
do not interrupt CPR to do so. One EMT should perform CPR while the other radios for
assistance. Continue CPR and reanalyze the patient’s cardiac rhythm every 2 minutes. If
indicated, deliver a single shock and immediately resume CPR, starting with chest
compressions. If the AED gives a no shock message, resume CPR, starting with chest
compressions. Continue CPR, rhythm analysis every 2 minutes, and defibrillation (if indicated),
until backup arrives or the patient starts to move.!
!
=====================================!
!
Your assessment of a middle-aged female with chest pressure reveals that she is confused,
diaphoretic, and has a blood pressure of 70/50 mm Hg. In caring for this patient, it is MOST
important for you to:!
!
A:!
prepare for immediate transport to the hospital.!
B:!
request an ALS unit to respond to the scene.!
C:!
reassess her vital signs at least every 5 minutes.!
D:!
assist her with her nitroglycerin if she has any.!
You selected A; This is correct!!
!
Reason:!
!
Your patient is in shock, which is most likely the result of heart failure (cardiogenic shock).
Immediate transport to the closest appropriate hospital is critical. Assess for and manage
problems with airway, breathing, and circulation, and then get on the road! She needs definitive
care that can only be provided at the hospital. Reassess her vital signs at least every 5 minutes,
but do it en route! If you can rendezvous with an ALS unit en route, do so. However, you should
not remain at the scene to wait for them. Nitroglycerin is clearly contraindicated for this patient;
her BP is dangerously low.!
!
=====================================!
!
By which of the following mechanisms does nitroglycerin relieve cardiac-related chest pain or
discomfort?!
!
A:!
Vasoconstriction and increased myocardial workload!
B:!
Vasodilation and decreased myocardial oxygen supply!
C:!
Vasoconstriction and increased cardiac oxygen demand!
D:!
Vasodilation and increased myocardial oxygen supply!
You selected A; The correct answer is D;!
!
Reason:!
!
Nitroglycerin is a smooth muscle relaxant. Smooth muscle is found within the walls of the blood
vessels. Nitroglycerin causes vasodilation, including dilation of the coronary arteries, which in
turn increases the flow of oxygenated blood to the heart and reduces myocardial workload.
However, care must be taken when administering nitroglycerin to a patient. Because of its
vasodilatory effects, nitroglycerin can cause hypotension; therefore, it should not be given to
patients with a systolic blood pressure of less than 100 mm Hg.!
!
=====================================!
!
Which of the following is the MOST detrimental effect that tachycardia can have on a patient
experiencing a cardiac problem?!
!
A:!
Increased stress and anxiety!
B:!
Increased oxygen demand!
C:!
Increased blood pressure!
D:!
Decreased cardiac functioning!
You selected A; The correct answer is B;!
!
Reason:!
!
As the heart beats faster, it consumes and demands more oxygen. Tachycardia can be
extremely detrimental to the patient with a compromised heart that is already deprived of
oxygen. Keeping the patient calm cannot be overemphasized. The more stressed and anxious
the patient gets, the faster his or her heart will beat.!
!
=====================================!
!
Treatment for a patient with congestive heart failure and shortness of breath may include:!
!
A:!
prophylactic suctioning of the airway.!
B:!
up to three doses of sublingual nitroglycerin.!
C:!
supine positioning and elevation of the legs.!
D:!
hyperventilation with a bag-mask device.!
You selected A; The correct answer is B;!
!
Reason:!
!
Treatment for patients with congestive heart failure (CHF) includes supplemental oxygen as
needed to maintain an oxygen saturation equal to or greater than 94%, continuous positive
airway pressure (CPAP), ventilatory assistance with a bag-mask device if needed (do NOT
hyperventilate the patient), placing the patient in an upright or sitting position to facilitate
breathing (a supine position will clearly make it more difficult to breathe), monitoring the patient's
vital signs, and transporting to the hospital without delay. Suction the airway only if there are
secretions in the mouth; prophylactic suctioning is not indicated. Nitroglycerin (NTG) may be of
value if the patient is not hypotensive and he or she has the medication prescribed to him or her.
As a vasodilator, NTG causes systemic venous pooling of blood, which reduces the amount of
blood returned to the heart (preload), and thus, the amount of blood available to backup in the
lungs. Follow your local protocols or contact medical control as needed regarding the use of
NTG for patients with CHF.!
!
=====================================!
!
The MOST important initial treatment for a patient whose cardiac arrest was witnessed is:!
!
A:!
rapid transport.!
B:!
high-quality CPR.!
C:!
cardiac drug therapy.!
D:!
defibrillation.!
You selected A; The correct answer is B;!
!
Reason:!
!
Regardless of whether a patient's cardiac arrest is witnessed or unwitnessed, the single most
important initial treatment is high-quality CPR. Delays in performing CPR have been clearly
linked to poor patient outcomes. After CPR has been initiated, apply the AED as soon as it is
available. Cardiac drug therapy and rapid transport enhance the patient's chance of survival, but
are useless without minimally-interrupted, high-quality CPR.!
!
=====================================!
!
The quickest way to reduce cardiac ischemia in a patient experiencing an acute coronary
syndrome is to:!
!
A:!
sit or lay the patient down.!
B:!
elevate the patient’s legs.!
C:!
give supplemental oxygen.!
D:!
keep the patient warm.!
You selected A; This is correct!!
!
Reason:!
!
Ischemia is defined as a relative lack of oxygen. In other words, relative to the body’s demand
for oxygen, its supply is reduced. The quickest way to reduce ischemia in a patient with an
acute coronary syndrome, or ACS (eg, unstable angina, acute myocardial infarction) is to
reduce the heart’s demand for oxygen; simply having the patient cease any exertion and placing
him or her in a sitting or lying position will quickly and effectively accomplish this. Although
supplemental oxygen increases the oxygen content of the blood and is an important treatment
for any patient with ACS, it does not reduce the body’s demand for oxygen.!
!
=====================================!
!
The middle, muscular layer of the heart is called the:!
!
A:!
epicardium.!
B:!
endocardium.!
C:!
pericardium.!
D:!
myocardium.!
You selected A; The correct answer is D;!
!
Reason:!
!
The heart has four layers. The inner layer is called the endocardium, the middle layer is
composed of muscle and is called the myocardium (myo = muscle), and the outer layer of the
heart itself is called the epicardium. The pericardium, which is a thin, fibrous membrane,
encapsulates the entire heart.!
!
=====================================!
!
After attaching the AED and pushing the analyze button on an adult patient in cardiac arrest, the
AED states that a shock is advised. What cardiac rhythm is the patient MOST likely in?!
!
A:!
Pulseless electrical activity!
B:!
Asystole!
C:!
Ventricular tachycardia!
D:!
Ventricular fibrillation!
You selected A; The correct answer is D;!
!
Reason:!
!
Ventricular fibrillation (V-Fib) is the most common initial cardiac dysrhythmia in adult cardiac
arrest patients, occurring in up to 75% of all cases. V-Fib is a chaotic quivering of the heart
muscle that does not produce a pulse and is due to a massive, uncontrolled electrical discharge
of the cardiac cells. The most effective treatment for V-Fib is defibrillation. Some patients are in
ventricular tachycardia (V-Tach) without a pulse, which is also treated with defibrillation.
Although asystole and pulseless electrical activity (PEA) do not produce a pulse, they are not
treated with defibrillation.!
!
=====================================!
!
Nitroglycerin is contraindicated in all of the following situations, EXCEPT:!
!
A:!
recent use of Cialis.!
B:!
the presence of a head injury.!
C:!
history of cardiac bypass surgery.!
D:!
systolic BP of 80 mm Hg.!
You selected A; The correct answer is C;!
!
Reason:!
!
Nitroglycerin (NTG) is a vasodilator drug used to relieve chest pain in patients with cardiac
compromise by dilating the coronary arteries and improving blood flow to the heart. Because of
its vasodilator effects, it should not be given to patients who have a systolic BP less than 100
mm Hg or to patients who have recently (within the past 24 to 48 hours) taken erectile
dysfunction (ED) drugs (eg, sildenafil [Viagra], vardenafil [Levitra], tadalafil [Cialis]). ED drugs
also cause vasodilation and may cause significant hypotension if given together with NTG. You
should also avoid NTG in patients with a head injury; dilation of the cerebral blood vessels may
worsen intracranial pressure caused by the head injury. By itself, a history of cardiac bypass
surgery does not contraindicate the use of NTG.!
!
=====================================!
!
Which of the following is the MOST appropriate response when a patient with chest pain asks
you if he or she is having a heart attack?!
!
A:!
I don’t know, but we will take good care of you.!
B:!
Yes, so I recommend going to the hospital.!
C:!
I believe you are, but only a physician can tell for sure.!
D:!
Probably not, but we should transport you to be safe.!
You selected A; This is correct!!
!
Reason:!
!
Patients experiencing chest pain often have a good idea about what is happening, so do not lie
or offer false reassurance. Conversely, do not tell the patient they are having a heart attack; this
can only be diagnosed by a physician, not an EMT in the field. If asked, “Am I having a heart
attack?” an appropriate response would be, “I don’t know for sure, but in case you are, we are
going to take good care of you.”!
!
=====================================!
!
Following administration of nitroglycerin to a man with crushing chest pressure, he experiences
a significant increase in his heart rate. This is MOST likely the result of:!
!
A:!
a cardiac dysrhythmia.!
B:!
coronary vasoconstriction.!
C:!
a drop in blood pressure.!
D:!
preexisting hypertension.!
You selected A; The correct answer is C;!
!
Reason:!
!
Nitroglycerin (NTG) is a vasodilator drug. It dilates not only the coronary arteries, but also other
arteries in the body. In some patients, NTG may cause a drop in blood pressure, especially if
they are taking other medications that cause vasodilation (eg, erectile dysfunction drugs [Viagra,
Levitra, Cialis]). In response to a drop in blood pressure, the nervous system attempts to
compensate by increasing the heart rate (tachycardia). Common side effects of NTG include a
headache, a burning sensation under the tongue, or a bitter taste in the mouth. Always assess
the patient’s vital signs, before and after administering nitroglycerin.!
!
=====================================!
!
The AED analyzes your pulseless and apneic patient's cardiac rhythm and advises that a shock
is NOT indicated. You should:!
!
A:!
open the patient's airway and check for breathing.!
B:!
reanalyze the cardiac rhythm for positive confirmation.!
C:!
resume CPR, starting with chest compressions.!
D:!
assess for a pulse for no more than 10 seconds.!
You selected A; The correct answer is C;!
!
Reason:!
!
If the AED advises "no shock," you should immediately resume CPR, starting with chest
compressions. Unless the patient starts to move or has other signs of life (ie, coughing),
stopping CPR to assess for a pulse should be avoided because it only causes an unnecessary
delay in performing chest compressions. The AED is a highly reliable device; if it advises that a
shock is or is not indicated, take its word for it! There is no need to reanalyze the patient's
cardiac rhythm; this only causes an unnecessary delay in defibrillation (if indicated) and chest
compressions.!
!
=====================================!
!
Common signs and symptoms of a hypertensive emergency include:!
!
A:!
vomiting without nausea and hemiparesis.!
B:!
chest discomfort, weak pulses, and cool skin.!
C:!
unequal pupils, irregular pulse, and pallor.!
D:!
ringing in the ears, headache, and epistaxis.!
You selected A; The correct answer is D;!
!
Reason:!
!
Although different sources cite various values, most agree that a hypertensive emergency exists
when the systolic blood pressure exceeds 180 mm Hg and the diastolic blood pressure exceeds
110 mm Hg. A hypertensive emergency also exists when the systolic BP suddenly rises and
produces signs and symptoms. One of the most common symptoms of a hypertensive
emergency is a severe headache. Other signs and symptoms include ringing in the ears
(tinnitus), epistaxis (nosebleed), bounding pulses, flushed skin (dry or moist), nausea and
vomiting, and dizziness. In severe cases, mental status changes may occur and the patient may
experience a sudden onset of pulmonary edema. Untreated hypertensive emergencies can lead
to a stroke or aortic dissection. By itself, a hypertensive emergency does not usually cause
unequal pupils or weakness to one side of the body (hemiparesis); however, these findings may
be observed if the patient experiences a stroke secondary to a hypertensive emergency.!
!
=====================================!
!
After assisting a patient with his or her prescribed nitroglycerin, you would NOT expect him or
her to experience a:!
!
A:!
pounding headache.!
B:!
syncopal episode.!
C:!
fizzing sensation under the tongue.!
D:!
burning sensation in the mouth.!
You selected A; The correct answer is B;!
!
Reason:!
!
Nitroglycerin (NTG) dilates the coronary arteries, thus increasing oxygen supply to the heart and
relieving a cardiac patient’s chest pain. Before assisting a patient with his or her prescribed
NTG, there are some predictable side effects you should make him or her aware of. Because
NTG dilates the blood vessels throughout the entire body, including those in the brain, a
pounding headache is common. A burning sensation in the mouth and/or fizzing sensation
under the tongue are also common side effects. Syncope (fainting), however, is not a normal
side effect of NTG, and suggests a substantial drop in blood pressure. If the patient becomes
dizzy or experiences a syncopal episode after being given NTG, place him or her in a supine
position. If necessary, elevate his or her legs to help increase blood pressure.!
!
=====================================!
!
The myocardium receives its blood supply from the coronary arteries that branch directly from
the:!
!
A:!
vena cava.!
B:!
aorta.!
C:!
right ventricle.!
D:!
left atrium.!
You selected A; The correct answer is B;!
!
Reason:!
!
The aorta, which is the largest artery in the human body, originates immediately from the left
ventricle where it branches into the coronary arteries. This allows the myocardium to receive
blood that has the highest concentration of oxygen. The superior and inferior venae cavae
return oxygen-poor blood from the systemic circulation back to the right atrium, where it is
pumped into the right ventricle. The left atrium receives freshly oxygenated blood from the
lungs.!
!
=====================================!
!
Which of the following questions would be MOST appropriate to ask when assessing a patient
with chest pain?!
!
A:!
B:!
C:!
What does the pain feel like?!
Is the pain worse when you take a deep breath?!
Would you describe the pain as sharp?!
D:!
Does the pain radiate to your arm?!
You selected A; This is correct!!
!
Reason:!
!
When questioning any patient about any type of pain, you should avoid asking leading
questions that can simply be answered yes or no. To obtain the most reliable assessment,
open-ended questions should be asked to allow the patient to describe the quality of the pain in
his or her own words.!
!
=====================================!
!
Which of the following statements regarding ventricular fibrillation (V-Fib) is correct?!
!
A:!
Loss of consciousness occurs within minutes after the onset of V-Fib.!
B:!
In V-Fib, the heart is not pumping any blood and the patient is pulseless.!
C:!
Any patient in V-Fib must receive CPR for 2 minutes prior to defibrillation.!
D:!
Patients in V-Fib should be defibrillated after every 60 seconds of CPR.!
You selected A; The correct answer is B;!
!
Reason:!
!
Ventricular fibrillation (V-Fib) is a disorganized, ineffective quivering of the heart muscle. No
blood is pumped through the body and the patient is pulseless. Loss of consciousness occurs
within seconds following the onset of V-Fib. Patients in V-Fib are treated with high-quality CPR
and defibrillation every 2 minutes if needed. When treating a patient in cardiac arrest, whether
the patient's arrest was witnessed or unwitnessed, begin immediate CPR and apply the AED as
soon as it is available.!
!
=====================================!
!
A 50-year-old man presents with crushing chest pain of sudden onset. He is diaphoretic,
apprehensive, and tachypneic. You should:!
!
A:!
perform a complete physical exam.!
B:!
obtain baseline vital signs.!
C:!
ask him if he takes nitroglycerin.!
D:!
apply supplemental oxygen.!
You selected A; The correct answer is D;!
!
Reason:!
!
All of the interventions and assessments listed in this question should be performed on a patient
who presents with chest pain, pressure, or discomfort. However, supplemental oxygen is
indicated for any patient with a potential cardiac problem and should be given as soon as
possible; this is especially true when the patient has potential respiratory involvement as well
(ie, dyspnea, tachypnea). Administer oxygen in a concentration sufficient to maintain an oxygen
saturation of 94% or greater. Aspirin (up to 325 mg) should also be administered as soon as
possible, unless the patient is allergic to it. After applying oxygen and administering aspirin, you
should perform a physical exam and obtain baseline vital signs. You would inquire about any
prescription medications the patient is taking (eg, NTG) during the SAMPLE history.!
!
=====================================!
!
Sudden cardiac arrest in the adult population MOST often is the result of:!
!
A:!
myocardial infarction.!
B:!
accidental electrocution.!
C:!
a cardiac arrhythmia.!
D:!
respiratory failure.!
You selected A; The correct answer is C;!
!
Reason:!
!
The most common cause of sudden cardiac arrest (SCA) in the adult population is a cardiac
arrhythmia—usually ventricular fibrillation—in up to 70% of cases. This fact underscores the
importance of early defibrillation to shock the heart back into a perfusing rhythm. Evidence has
shown that cardiac arrest—again, most often the result of an arrhythmia—occurs in up to 40%
of patients experiencing an acute myocardial infarction (AMI). The risk of cardiac arrest is
highest within the first few hours following the onset of an AMI. Respiratory failure is the most
common cause of cardiac arrest in children, not adults. Children generally have healthy hearts
and rarely experience cardiac arrest due to a primary cardiac event.!
!
=====================================!
!
You are treating a 60-year-old man in cardiac arrest. After delivering a shock with the AED and
performing CPR for 2 minutes, you achieve return of spontaneous circulation. Your next action
should be to:!
!
A:!
assess his airway and ventilatory status.!
B:!
provide rapid transport to the hospital.!
C:!
remove the AED and apply 100% oxygen.!
D:!
reanalyze his rhythm for confirmation.!
You selected A; This is correct!!
!
Reason:!
!
If return of spontaneous circulation (ROSC) occurs (eg, a palpable pulse is restored), your first
action should be to reassess the patient's airway and ventilatory status. If the patient remains
apneic, continue rescue breathing. If the patient is breathing adequately, administer high-flow
oxygen. After reassessing airway and breathing, and treating accordingly, you should prepare
for immediate transport. Because of the high risk that cardiac arrest can recur following
resuscitation, you should not remove the AED pads; simply turn the AED off instead. Analysis of
the patient's cardiac rhythm is not indicated because he now has a pulse.!
!
=====================================!
!
After applying the AED to an adult patient in cardiac arrest, you analyze her cardiac rhythm and
receive a shock advised message. Emergency medical responders, who arrived at the scene
before you, tell you that bystander CPR was not in progress upon their arrival. You should:!
!
A:!
notify medical control and request permission to cease resuscitation.!
B:!
perform CPR for 2 minutes and then defibrillate.!
C:!
deliver the shock as indicated followed immediately by CPR.!
D:!
detach the AED and prepare for immediate transport.!
You selected A; The correct answer is C;!
!
Reason:!
!
Information obtained at the scene—whether from emergency medical responders or family
members—is not always accurate. If the AED advises you to defibrillate, you should do so and
then immediately begin or resume CPR, starting with chest compressions. During CPR, ensure
adequate oxygenation and ventilation, minimize interruptions in chest compressions, obtain the
patient's medical history if possible, and contact medical control as needed. At no time during
resuscitative efforts should you detach the AED from the patient. Reanalyze the patient's
cardiac rhythm after 2 minutes of CPR and follow the voice prompts.!
!
=====================================!
!
Chest compression effectiveness is MOST effectively assessed by:!
!
A:!
measuring the systolic blood pressure during compressions.!
B:!
listening for a heartbeat with each compression.!
C:!
palpating for a carotid pulse with each compression.!
D:!
carefully measuring the depth of each compression.!
You selected A; The correct answer is C;!
!
Reason:!
!
When chest compressions are in progress, the most reliable method of determining their
effectiveness is to palpate for a carotid or femoral pulse. If compressions are of adequate depth
for the patient’s age, you should be able to feel a pulsation during each compression. It should
be noted that you may not be able to feel a pulse in severely hypovolemic patients, despite
adequately performed chest compressions.!
!
=====================================!
!
Which of the following patients would be the LEAST likely to present with classic signs and
symptoms of acute myocardial infarction?!
!
A:!
72-year-old female with diabetes and hypertension.!
B:!
55-year-old female with COPD and frequent infections.!
C:!
64-year-old male with renal disease and depression.!
D:!
59 year-old male with alcoholism and angina pectoris.!
You selected A; This is correct!!
!
Reason:!
!
Chest pain, pressure, or discomfort (usually lasting greater than 15 minutes) is present in the
majority of patients experiencing acute myocardial infarction (AMI). Other common signs and
symptoms include shortness of breath, nausea, and diaphoresis. However, elderly female
patients—especially those with diabetes—are more likely to present with atypical or unusual
signs and symptoms than any other patient population. Diabetic neuropathy—a degenerative
nerve condition associated with diabetes—results in decreased sensitivity to pain; therefore, the
patient may present without any pain or discomfort. Sometimes, the only presenting signs and
symptoms of AMI are generalized weakness, fatigue, or fainting.!
!
=====================================!
!
Tachycardia can be detrimental to a patient who is experiencing a cardiac problem because it
causes:!
!
A:!
a profound decrease in oxygen consumption.!
B:!
an associated increase in breathing difficulty.!
C:!
increased cardiac filling in between beats.!
D:!
increased cardiac oxygen usage and demand.!
You selected A; The correct answer is D;!
!
Reason:!
!
Many patients experiencing a cardiac problem are tachycardic (heart rate greater than 100
beats/min); others are bradycardic (heart rate less than 60 beats/min). As the heart beats faster,
it consumes, and therefore requires, more oxygen. This can be detrimental to the patient
because the heart is already deprived of oxygen (ischemia) and may not be able to
accommodate the increased need for oxygen; this may worsen the ischemia, potentially causing
further cardiac damage. A very fast heart rate (>150 beats/min) may cause hypotension due to a
decrease in cardiac output; it occurs because the ventricles are not adequately filling with blood
in between beats. Some patients with a cardiac problem may have difficulty breathing, which
may be the result of congestive heart failure and pulmonary edema; it is not caused by the
tachycardia itself.!
!
=====================================!
!
Aspirin is beneficial to patients experiencing an acute coronary syndrome because it:!
!
A:!
effectively relieves their chest pain.!
B:!
destroys the clot that is blocking a coronary artery.!
C:!
prevents a clot from getting larger.!
D:!
decreases cardiac workload by lowering the BP.!
You selected A; The correct answer is C;!
!
Reason:!
!
Early administration of baby aspirin (160 to 325 mg) to patients with acute coronary syndrome
(ACS) has clearly been shown to reduce mortality and morbidity. Aspirin (acetylsalicylic acid
[ASA]) prevents the clot in a coronary artery from getting larger by inhibiting platelet
aggregation; in other words, it makes the platelets less sticky, which means that they will have
less of a tendency to clump together. Aspirin does not relieve the chest pain or discomfort
associated with ACS, nor does it reduce blood pressure. Furthermore, aspirin does not destroy
the clot that is blocking a coronary artery; fibrinolytic (clot-buster) drugs actually destroy the clot.!
!
=====================================!
!
All of the following are components of the Cincinnati stroke scale, EXCEPT:!
!
A:!
pupil size.!
B:!
arm movement.!
C:!
speech pattern.!
D:!
facial droop.!
You selected A; This is correct!!
!
Reason:!
!
The Cincinnati Stroke Scale, which tests speech, facial droop, and arm drift, is a reliable tool
that should be used during your assessment of a patient suspected of having had a stroke. To
test speech, ask the patient to repeat a simple phrase; he or she should be able to repeat the
phrase without slurred speech. If the patient cannot repeat the phrase, or repeats it with slurred
speech, a stroke should be assumed. To test facial movement, ask the patient to smile and
show his or her teeth; both sides of the face should move symmetrically (equally). If only one
side of the face is moving well (facial droop), a stroke should be assumed. To test arm
movement, ask the patient to hold both arms in front of his or her body, palms up, with eyes
closed and without moving. Over the next 10 seconds, watch the patient’s hands. If one arm
drifts down toward the ground, you know that side is weak. You should assess pupillary size,
equality, and reactivity in any patient with a neurological disorder; however, this is not a part of
the Cincinnati Stroke Scale.!
!
=====================================!
!
After delivering one shock with the AED and performing 2 minutes of CPR on a woman in
cardiac arrest, you reanalyze her cardiac rhythm and receive a no shock advised message. This
means that:!
!
A:!
her rhythm has deteriorated to asystole.!
B:!
she is not in a shockable rhythm.!
C:!
the first shock restored a rhythm and pulse.!
D:!
she has electrical activity but no pulse.!
You selected A; The correct answer is B;!
!
Reason:!
!
If the AED gives a no shock advised message, it has determined that the patient is not in a
shockable rhythm (eg, V-Fib, pulseless V-Tach). It does not indicate that the patient has a pulse,
nor does it indicate that a normal cardiac rhythm has been restored. The AED does not
distinguish pulseless electrical activity (PEA) from asystole; it only recognizes them as
nonshockable. PEA is a condition in which organized cardiac electrical activity is present despite
the absence of a pulse. Asystole is the absence of all cardiac electrical and mechanical activity.
If the AED gives a no shock advised message, immediately resume CPR, starting with chest
compressions, until ALS arrives or the patient starts to move.!
!
=====================================!
!
Switching compressors during two-rescuer CPR:!
!
A:!
is performed after every 10 to 20 cycles of adult CPR.!
B:!
should take no more than 15 seconds to accomplish.!
C:!
is only necessary if the compressor becomes fatigued.!
D:!
should occur every 2 minutes throughout the arrest.!
You selected A; The correct answer is D;!
!
Reason:!
!
Rescuer fatigue may lead to inadequate chest compression rate and/or depth. Fatigue is
common after 1 minute of CPR, although the rescuer may not recognize it for 5 minutes or
longer. Therefore, compressors should be changed every 2 minutes (after 5 cycles of CPR at a
30:2 ratio) throughout the resuscitation attempt. If the compressor is not switched until he or she
recognizes the fatigue, the patient has likely been without effective chest compressions for at
least 4 or 5 minutes. In general, interruptions in CPR should be infrequent and should not
exceed 10 seconds. However, every effort should be made to switch compressors in less than 5
seconds.!
!
=====================================!
!
Which of the following is a common side effect of nitroglycerin?!
!
A:!
Headache!
B:!
Nausea!
C:!
Hypertension!
D:!
Anxiety!
You selected A; This is correct!!
!
Reason:!
!
Because nitroglycerin (NTG) causes vasodilation, including the vessels within the brain,
cerebral blood flow increases following the administration of NTG. This often causes a pounding
headache for the patient. As uncomfortable as it is for the patient, headaches are a common
and expected side effect of the drug. The vasodilatory effects of nitroglycerin could result in
hypotension; therefore, the patient’s blood pressure should be carefully monitored. Nausea and
anxiety are common symptoms of acute coronary syndrome; they are not common side effects
of nitroglycerin.!
!
=====================================!
!
You have analyzed a cardiac arrest patient's rhythm three times with the AED, separated by 2minute cycles of CPR, and have received no shock messages each time. You should:!
!
A:!
continue CPR and transport at once.!
B:!
request a paramedic unit at the scene.!
C:!
remove the AED and continue CPR.!
D:!
consider terminating resuscitation.!
You selected A; This is correct!!
!
Reason:!
!
Although protocols vary from system to system, it is generally agreed that if you receive three
consecutive no shock messages, separated by 2-minute cycles of CPR, you should continue
CPR and transport at once; it is unlikely that the patient will convert to a shockable rhythm (eg,
V-Fib, pulseless V-Tach). En route, coordinate a rendezvous with a paramedic unit if possible;
waiting at the scene would only delay further treatment. The decision to terminate resuscitative
efforts is made by a physician, and in some cases, a paramedic, after adequately performed
BLS and ALS have proven unsuccessful.!
!
=====================================!
!
A 60-year-old man presents with chest pain and difficulty breathing. He is pale, diaphoretic, and
in severe pain. As your partner applies supplemental oxygen, you assess his vital signs. His
blood pressure is 180/90 mm Hg, pulse is 110 beats/min and irregular, and respirations are 24
breaths/min and labored. You ask him if has taken any nitroglycerin and he tells you that he
does not have any but his wife does. You should:!
!
A:!
have him swallow up to four enteric-coated aspirin, continue oxygen therapy, and
promptly transport him to the hospital.!
B:!
contact medical control and request permission to assist the patient with up to three
doses of his wife’s nitroglycerin.!
C:!
complete your focused physical examination and prepare the patient for immediate
transport to an appropriate hospital.!
D:!
transport at once, apply the AED in case he develops cardiac arrest, and monitor his
vital signs en route to the hospital.!
You selected A; The correct answer is C;!
!
Reason:!
!
If a patient with suspected cardiac compromise does not have prescribed nitroglycerin (NTG),
complete your focused physical examination, continue oxygen therapy, and transport
immediately. Do not administer, or request to administer, any medication that is not prescribed
to the patient. If the patient develops cardiac arrest, apply the AED and follow its voice prompts.
Do not apply the AED to any patient who is not in cardiac arrest. If your protocols allow you to
administer aspirin, give up to 325 mg of chewable baby aspirin (have the patient chew the
aspirin before swallowing it). Enteric-coated aspirin (aspirin that is coated to prevent stomach
upset) is intended to be swallowed without chewing. It takes too long to dissolve, whereas
chewable aspirin has a much faster effect.!
!
=====================================!
!
Which of the following is MOST indicative of a primary cardiac problem?!
!
A:!
Irregular pulse!
B:!
Tachypnea!
C:!
Sudden fainting!
D:!
Tachycardia!
You selected A; This is correct!!
!
Reason:!
!
An irregular pulse signifies an abnormality within the electrical conduction system of the heart.
Tachycardia, sudden fainting (syncope), and tachypnea (rapid breathing) can indicate many
things other than cardiac problems, such as shock, heat-related problems, and diabetic
complications. You should always consider the possibility of a cardiac problem in a patient with
an irregular pulse.!
!
=====================================!
!
Freshly oxygenated blood returns to the heart via the:!
!
A:!
pulmonary artery.!
B:!
pulmonary vein.!
C:!
aorta.!
D:!
vena cavae.!
You selected A; The correct answer is B;!
!
Reason:!
!
The pulmonary vein is the only vein that carries oxygen-rich blood. It carries blood from the
lungs back to the left atrium. All other veins in the human body, including the vena cavae, carry
deoxygenated blood back to the heart. The aorta is the largest artery in the body and branches
immediately from the left ventricle, carrying freshly oxygenated blood to the rest of the body. The
pulmonary artery carries deoxygenated blood from the right ventricle to the lungs for
reoxygenation.!
!
=====================================!
!
Which of the following questions would be the MOST effective in determining if a patient's chest
pain radiates away from his or her chest?!
!
A:!
Is there anything that makes the pain better or worse?!
B:!
Does the pain stay in your chest or move anywhere else?!
C:!
Is there any other part of your body where you have pain?!
D:!
Do you also have pain in your arm, jaw, or back?!
You selected A; The correct answer is B;!
!
Reason:!
!
When assessing a patient with any type of pain, you should avoid asking leading questions;
instead, ask open-ended questions whenever possible. For example, instead of asking the
patient if his or her pain is dull, crushing, or sharp, ask him or her to describe the pain using his
or her own words. Patients with radiating pain often state that the pain moves or travels away
from its point of origin, with pain in between point A and point B. Patients with referred pain
complain of pain in more than one location, without a trail of pain in between. You should also
ask the patient if anything makes the pain worse (provokes) or better (palliates).!
!
=====================================!
!
You should suspect that your patient has pulmonary edema if he or she:!
!
A:!
has swollen feet and ankles.!
B:!
has a dry, nonproductive cough.!
C:!
is hypertensive and tachycardic.!
D:!
cannot breathe while lying down.!
You selected A; The correct answer is D;!
!
Reason:!
!
Pulmonary edema is often caused by failure of the left side of the heart. When the patient is
lying down, he or she experiences worsened difficulty breathing (orthopnea) because more
blood backs up in the lungs. Patients with severe pulmonary edema often produce pink, frothy
sputum when they cough; this is another sign of blood backing up in the lungs. A dry,
nonproductive cough is not common. Hypertension and tachycardia are common in patients
with pulmonary edema; however, many other conditions can cause these findings. Swelling of
the feet and ankles is commonly seen in patients with right heart failure, and occurs when blood
backs up beyond the right atrium; it is not a common sign of left heart failure and pulmonary
edema.!
!
=====================================!
!
While assessing a patient with chest pain, you note that his pulse is irregular. This indicates:!
!
A:!
abnormalities in the heart’s electrical conduction system.!
B:!
acute myocardial infarction or angina pectoris.!
C:!
high blood pressure that is increasing cardiac workload.!
D:!
a dysfunction in the left side of the patient’s heart!
You selected A; This is correct!!
!
Reason:!
!
An irregular pulse indicates abnormalities in the electrical conduction system of the heart. The
electrical conduction system, beginning with the sinoatrial node as the primary pacemaker, is
responsible for initiating the electrical impulses that stimulate the myocardium to contract. An
irregular pulse could indicate potentially lethal arrhythmias that could result in cardiac arrest.
You should document an irregular pulse and report this important finding to the emergency
department.!
!
=====================================!
!
Which of the following interventions would the EMT be the LEAST likely to perform while
attempting to resuscitate a cardiac arrest patient?!
!
A:!
Insertion of a supraglottic airway device.!
B:!
Rhythm analysis with the AED!
C:!
Assisting a paramedic with intubation!
D:!
Ventilation with a bag-mask device!
You selected A; This is correct!!
!
Reason:!
!
The insertion of advanced airway devices (eg, endotracheal [ET] tube, multilumen airway,
supraglottic airway) is generally outside the EMT’s scope of practice. However, he or she may
be asked to assist a paramedic in the placement of such devices. For example, the paramedic
may ask the EMT to retrieve the appropriate equipment or preoxygenate the patient before he
or she inserts the device. Operation of the AED and ventilating with a bag-mask device are
within the EMT’s scope of practice.!
!
=====================================!
!
When treating a patient with chest pain, pressure, or discomfort, you should first:!
!
A:!
place the patient in a position of comfort.!
B:!
administer supplemental oxygen.!
C:!
assess the blood pressure and give nitroglycerin.!
D:!
request an ALS ambulance response to the scene.!
You selected A; This is correct!!
!
Reason:!
!
An important aspect of treating a patient with chest pain, pressure, or discomfort is to ensure
that the patient is in a comfortable position. Most of the time, the patient will already be in this
position upon your arrival. A position of comfort will aid in minimizing anxiety, which in turn
decreases cardiac oxygen consumption and demand. After ensuring that the patient is in a
comfortable position, administer supplemental oxygen in a concentration sufficient to maintain
an SpO2 of greater than 94%. Following your assessment, if you feel that ALS support is
needed, you should request it. If the patient has prescribed, unexpired nitroglycerin; the systolic
blood pressure is greater than 100 mm Hg; and the patient has not taken the maximum of three
doses, you should contact medical control to obtain permission to assist the patient in taking the
nitroglycerin.!
!
=====================================!
!
While assessing a man who is complaining of chest pain, he suddenly becomes unresponsive.
You should:!
!
A:!
apply the AED.!
B:!
assess for a pulse.!
C:!
assess for breathing.!
D:!
open the airway.!
You selected A; The correct answer is C;!
!
Reason:!
!
Whether a patient is found unresponsive or becomes unresponsive in your presence, you
should immediately assess his or her breathing. Quickly look at the chest for visible rise. If the
patient is not breathing (or only has agonal gasps), you should assess for a pulse. If the patient
has a pulse, open the airway and begin rescue breathing. If the patient does not have a pulse,
begin CPR (starting with chest compressions) and apply the AED as soon as possible.!
!
=====================================!
!
The wall that separates the left and right sides of the heart is called the:!
!
A:!
mediastinum.!
B:!
carina.!
C:!
pericardium.!
D:!
septum.!
You selected A; The correct answer is D;!
!
Reason:!
!
The septum is the wall that separates the left and right sides of the heart. There is a septum for
both the atria and the ventricles. The carina is the bifurcation point of the trachea, and the
mediastinum is the space between the lungs in which the heart, great vessels, and a portion of
the esophagus lie. The pericardium is the sac that surrounds the heart and contains pericardial
fluid.!
!
=====================================!
!
After restoring a pulse in a cardiac arrest patient, you begin immediate transport. While en route
to the hospital, the patient goes back into cardiac arrest. You should:!
!
A:!
begin CPR and proceed to the hospital.!
B:!
analyze the patient’s rhythm with the AED.!
C:!
tell your partner to stop the ambulance.!
D:!
contact medical control for further advice.!
You selected A; The correct answer is C;!
!
Reason:!
!
If you restore a pulse in a cardiac arrest patient, the AED pads should remain attached to the
patient's chest during transport in case cardiac arrest recurs en route to the hospital. You
should, however, turn the AED off or disconnect the pads from the AED. If cardiac arrest recurs,
you should immediately tell your partner to stop the ambulance and assist you as you begin
CPR. Remember that the AED will not analyze the cardiac rhythm if the patient is moving. Once
your partner is available to assist, you should analyze the cardiac rhythm, defibrillate if
indicated, and immediately resume CPR. Contact medical control as soon as possible, but not
before performing CPR and defibrillation.!
!
=====================================!
!
Which of the following types of stroke would MOST likely present with a sudden, severe
headache?!
!
A:!
Thrombotic!
B:!
Embolic!
C:!
Ischemic!
D:!
Hemorrhagic!
You selected A; The correct answer is D;!
!
Reason:!
!
A stroke occurs when blood flow to a portion of the brain is interrupted. There are two types of
stroke: ischemic and hemorrhagic, both of which are acute events. Hemorrhagic stroke is
caused by a ruptured artery in the brain, resulting in intracerebral bleeding, cerebral ischemia,
and increased intracranial pressure. A classic feature of a hemorrhagic stroke is a sudden,
severe headache (indicates cerebral artery rupture) that is followed by rapid deterioration of the
patient’s condition. By contrast, ischemic stroke (accounts for approximately 80% of all strokes)
occurs when a clot occludes a cerebral artery. If the clot forms locally, it is referred to as a
thrombotic stroke; if the clot travels to the brain from another part of the body, it is referred to as
an embolic stroke. Acute ischemic stroke typically presents with a sudden onset of confusion,
facial droop, slurred speech, and weakness to one side of the body. A headache is uncommon
in patients with acute ischemic stroke.!
!
=====================================!
!
Which of the following describes the MOST appropriate method of performing chest
compressions on an adult patient in cardiac arrest?!
!
A:!
Compress the chest at least 2", allow full recoil of the chest after each compression,
minimize interruptions in chest compressions!
B:!
Do not interrupt chest compressions for any reason, compress the chest no more than 1
1/2”, allow partial recoil of the chest after each compression!
C:!
Allow full recoil of the chest after each compression, compress the chest to a depth of
2”, deliver compressions at a rate of at least 80/min!
D:!
Minimize interruptions in chest compressions, provide 70% compression time and 30%
relaxation time, deliver compressions at a rate of 100/min!
You selected A; This is correct!!
!
Reason:!
!
Effective chest compressions are essential for providing blood flow during CPR. To perform
adequate chest compressions, the EMT should “push hard and push fast.” Compress the chest
of an adult at a rate of at least 100 compressions/min to a depth of at least 2”. When performing
chest compressions on an infant or child, compress the chest at least one third the depth of the
chest (about 1 1/2" for infants, about 2" for children). Allow the chest to fully recoil after each
compression, and allow equal time for compression and relaxation. Minimize interruptions in
CPR to 10 seconds or less. Obviously, chest compressions must be paused when using the
AED to analyze the patient's cardiac rhythm or defibrillate and when assessing for a pulse.!
!
=====================================!
!
A middle-aged female with a history of hypertension and high cholesterol complains of chest
discomfort. She asks you to take her to the hospital where her personal physician practices,
which is 15 miles away. Her blood pressure is 130/70 mm Hg, pulse is 84 beats/min and regular,
and respirations are 18 breaths/min and unlabored. Which of the following actions is clearly
NOT appropriate for this patient?!
!
A:!
Giving oxygen via nasal cannula!
B:!
Contacting her physician via phone!
C:!
Allowing her to walk to the ambulance!
D:!
Taking her to her choice hospital!
You selected A; The correct answer is C;!
!
Reason:!
!
You should NEVER allow a patient with a possible cardiac problem to walk to the ambulance.
This causes exertion, which increases cardiac oxygen consumption and demand and could
worsen his or her condition. Give the patient oxygen in a concentration sufficient to maintain his
or her oxygen saturation equal to or greater than 94%. In general, you should transport patients
to the hospital of their choice. However, transport to a closer hospital should be considered if
you believe the patient is unstable or is at high risk for becoming unstable. If necessary, consult
with the patient’s physician via phone to determine if he or she thinks the patient should go to a
closer hospital.!
!
=====================================!
!
Shortly after assisting a 60-year-old woman with her second nitroglycerin treatment, she tells
you that she is lightheaded and feels like she is going to faint. Her symptoms are MOST likely
due to:!
!
A:!
a drop in her blood sugar.!
B:!
an irregular heartbeat.!
C:!
nervousness and anxiety.!
D:!
low blood pressure.!
You selected A; The correct answer is D;!
!
Reason:!
!
Nitroglycerin (NTG) is a vasodilator; as such, it may cause a drop in blood pressure
(hypotension) in some patients. Signs and symptoms of hypotension include dizziness,
lightheadedness, and fainting (syncope), among others. For this reason, you should always
assess a patient’s blood pressure before and after administering nitroglycerin. If the patient’s
systolic BP is less than 100 mm Hg, NTG should not be given. The patient is probably nervous
and anxious, and may even have an irregular heartbeat; however, hypotension is a more likely
cause of her symptoms. NTG does not affect a patient’s blood sugar level.!
!
=====================================!
!
You arrive at the scene shortly after a 55-year-old man collapsed. Two bystanders are
performing CPR. Your FIRST action should be to:!
!
A:!
check the effectiveness of the CPR in progress.!
B:!
insert an oropharyngeal airway and continue CPR!
C:!
stop CPR so you can assess breathing and pulse.!
D:!
attach the AED and analyze his cardiac rhythm.!
You selected A; The correct answer is C;!
!
Reason:!
!
Upon arriving at a scene where bystander CPR is in progress, you must first confirm that the
patient is indeed apneic and pulseless and needs CPR. Bystanders who are not properly trained
often perform CPR on patients who do not need it. After confirming cardiac arrest, you should
resume CPR and attach the AED as soon as possible.!
!
=====================================!
!
After administering nitroglycerin to a patient with chest discomfort, it is MOST important for you
to:!
!
A:!
find out how long the discomfort has been present.!
B:!
place the patient supine and elevate his or her legs.!
C:!
ask the patient if the discomfort has improved.!
D:!
reassess his or her blood pressure within 5 minutes.!
You selected A; The correct answer is D;!
!
Reason:!
!
Nitroglycerin (NTG) relaxes the muscle of blood vessel walls, dilates the coronary arteries,
increases blood flow and the supply of oxygen to the heart muscle (myocardium), and
decreases the workload of the heart. NTG also dilates blood vessels in other parts of the body,
potentially resulting in hypotension. For this reason, you should reassess the patient’s blood
pressure within 5 minutes after each dose of NTG. If the systolic blood pressure is less than 100
mm Hg, do not give any more NTG. Position him or her supine and elevate his or her legs if
needed. Asking the patient if his or her chest pain or discomfort has improved following NTG
helps you determine if the drug is working and whether additional dosing is needed; however,
detecting hypotension is clearly more important. You should determine when the chest pain or
discomfort began during the focused history, which is typically performed before assisting a
patient with his or her prescribed NTG.!
!
=====================================!
!
You assess an unresponsive 65-year-old man and find that he is apneic and pulseless. The
patient's wife tells you that he has an automatic implanted cardioverter/defibrillator. After
initiating CPR, you should:!
!
A:!
avoid using the AED because the implanted defibrillator is more effective.!
B:!
ask the wife why and when he had the automatic defibrillator implanted.!
C:!
deactivate the implanted defibrillator by running a magnet over it.!
D:!
apply the AED as soon as possible and analyze his cardiac rhythm.!
You selected A; The correct answer is D;!
!
Reason:!
!
Some patients who are at high risk for sudden cardiac arrest due to ventricular fibrillation (V-Fib)
have a small automatic implanted cardioverter/defibrillator (AICD). The AICD attaches directly to
the heart and continuously monitors the cardiac rhythm, delivering a shock if V-Fib or another
lethal dysrhythmia is detected. Regardless of whether the patient has an AICD, he or she should
be treated like all other cardiac arrest patients. Perform CPR and use the AED as usual;
however, you should ensure that the AED pads are at least 1" away from the implanted device.
Generally, the electricity from the AICD is so low that it will have no effect on rescuers and
therefore should not be of concern to you. Do not deactivate an implanted AICD, especially if it
is working and delivering shocks as it is supposed to. When treating a cardiac arrest patient who
has an AICD, your priority is to provide CPR and defibrillate with the AED if indicated, not to
determine when and why the AICD was implanted.!
!
=====================================!
!
Sudden cardiac arrest in the adult population is MOST often the result of:!
!
A:!
myocardial infarction.!
B:!
an acute stroke.!
C:!
a cardiac dysrhythmia.!
D:!
respiratory failure.!
You selected A; The correct answer is C;!
!
Reason:!
!
Most cases of sudden cardiac arrest (SCA) in the adult (70% to 75%) are the result of a cardiac
dysrhythmia, most commonly ventricular fibrillation (V-Fib). This fact underscores the
importance of early defibrillation. Stroke, respiratory failure, and myocardial infarction can all
cause cardiac arrest in the adult, but a cardiac dysrhythmia is more common.!
!
=====================================!
!
A patient reports pain in the upper midabdominal area. This region of the abdomen is called the:!
!
A:!
B:!
C:!
mediastinum.!
epigastrium.!
peritoneum.!
D:!
retroperitoneum.!
You selected A; The correct answer is B;!
!
Reason:!
!
The mid-upper region of the abdomen is referred to as the epigastrium because of its location
over the stomach (epi = upon, gastric = stomach). This is a common site of pain or discomfort in
patients experiencing a cardiac problem, which frequently causes them to attribute their pain or
discomfort to indigestion.!
!
=====================================!
!
Which of the following is an abnormal finding when using the Cincinnati stroke scale to assess a
patient who presents with signs of a stroke?!
!
A:!
One arm drifts down compared with the other side.!
B:!
The patient’s face is symmetrical when he or she smiles.!
C:!
One of the pupils is dilated and does not react to light.!
D:!
Both arms drift slowly and equally down to the patient’s side.!
You selected A; This is correct!!
!
Reason:!
!
The Cincinnati Stroke Scale is used to assess patients suspected of experiencing a stroke. It
consists of three tests: speech, facial droop, and arm drift. Abnormality in any one of these
areas indicates a high probability of stroke. To test arm drift, ask the patient to hold both arms in
front of his or her body, palms facing upward, with eyes closed and without moving. Over the
next 10 seconds, observe the patient’s arms. If one arm drifts down toward the ground, you
know that side is weak; this is an abnormal finding. To test for facial droop, have the patient
smile, showing his or her teeth. The face should be symmetrical (both sides of the face should
move equally). If only one side of the face moves well, you know that something is wrong with
the part of the brain that controls the facial muscles. You should assess the pupils of a patient
with a suspected stroke; however, this is not a component of the Cincinnati Stroke Scale.!
!
=====================================!
!
A 50-year-old man's implanted defibrillator has fired twice within the last hour. He is conscious
and alert and complains of a "sore chest." Further assessment reveals that his chest pain is
reproducible to palpation and is localized to the area of his implanted defibrillator. In addition to
supplemental oxygen, treatment for him should include:!
!
A:!
up to three doses of nitroglycerin and prompt transport.!
B:!
application of the AED and transport to the hospital.!
C:!
prompt transport with continuous monitoring en route.!
D:!
deactivating his defibrillator by running a magnet over it.!
You selected A; The correct answer is C;!
!
Reason:!
!
Patients who are high risk for lethal cardiac dysrhythmias (ie, V-Fib, V-Tach) may have an
automated implantable cardioverter/defibrillator (AICD). This small device is usually implanted in
the upper left chest, just below the left clavicle. The AICD detects cardiac dysrhythmias and
rapidly delivers a shock. When treating a patient whose AICD has fired, you should determine
the number of times the device fired, administer supplemental oxygen as needed, obtain vital
signs, and transport to the hospital with continuous monitoring en route. Application of the AED
is not indicated; however, if the patient develops cardiac arrest, you should use the AED as you
normally would (remember to apply the pads at least 1" away from the implanted device). The
pain that the patient is experiencing, which is reproducible and localized near his AICD, is likely
musculoskeletal pain as the result of his AICD shocking him; therefore, nitroglycerin is not
indicated. Because the AICD works so quickly (much faster than you can apply an AED), you
should not make any attempt to deactivate it. !
!
=====================================!
!
You should be MOST suspicious that a patient with chest pressure has an underlying cardiac
problem if his or her pulse is:!
!
A:!
irregular.!
B:!
slow.!
C:!
rapid.!
D:!
bounding.!
You selected A; This is correct!!
!
Reason:!
!
Of the choices listed, an irregular pulse should make you the most suspicious that a patient with
chest pain, pressure, or discomfort has an underlying cardiac problem. An irregular pulse
indicates a cardiac dysrhythmia (abnormal cardiac rhythm), which may be a precursor to cardiac
arrest. A fast pulse (tachycardia), slow pulse (bradycardia), or bounding pulse can be caused by
numerous conditions, not all of which are cardiac related.!
!
=====================================!
!
How can you help maximize cardiac output during CPR?!
!
A:!
Compress the chest at a rate of no more than 100/min!
B:!
Deliver rescue breaths until the chest expands widely!
C:!
Ventilate the patient through an advanced airway device!
D:!
Allow the chest to fully recoil in between compressions!
You selected A; The correct answer is D;!
!
Reason:!
!
Cardiac output is the amount of blood ejected from the left ventricle per minute. Bearing in mind
that even the best performed CPR produces only between 25% and 30% of what the patient's
cardiac output would otherwise be, there are several actions that you must take to help
maximize this. Allowing the chest to fully recoil in between compressions will help draw blood
back to the heart; if more blood returns to the heart, more blood can be pumped from the heart
with chest compressions. Delivering each rescue breath over a period of 1 second, just enough
to produce visible chest rise, will also help maximize cardiac output. If ventilations are given to
fast or too forcefully, intrathoracic pressure will increase, resulting in a decrease in the amount
of blood that returns to the heart; as a result, cardiac output will decrease. Ventilations are
delivered no differently if an advanced airway device (ie, ET tube, multilumen airway,
supraglottic airway) has been inserted. Deliver chest compressions at a rate of at least 100 per
minute to a depth of at least 2" (at least one third the depth of the chest in infants and children).!
!
=====================================!
!
In addition to chest pain or discomfort, a patient experiencing an acute coronary syndrome
would MOST likely present with:!
!
A:!
severe projectile vomiting and flushed skin.!
B:!
ashen skin color, diaphoresis, and anxiety.!
C:!
profound cyanosis, dry skin, and a headache.!
D:!
irregular breathing and low blood pressure.!
You selected A; The correct answer is B;!
!
Reason:!
!
Chest pain, pressure, or discomfort is the most common symptom of acute coronary syndrome,
or ACS (eg, unstable angina, acute myocardial infarction); it occurs in approximately 80% of
cases. Patients with ACS are usually anxious and may have a feeling of impending doom.
Nausea and vomiting are common complaints; however, projectile vomiting, which is typically
associated with increased intracranial pressure, is uncommon. The skin is often ashen gray and
clammy (diaphoretic) because of poor cardiac output and decreased perfusion. Less commonly,
the patient’s skin is cyanotic. Respirations are usually unlabored unless the patient has
congestive heart failure, in which case respirations are rapid and labored; irregular breathing,
however, is not common. Blood pressure may fall as a result of decreased cardiac output;
however, most patients will have a normal or elevated blood pressure. If the patient complains
of a headache, it is usually a side effect of the nitroglycerin they took before your arrival; ACS
itself usually does not cause a headache.!
!
=====================================!
!
Which of the following structures is the primary pacemaker, which sets the normal rate for the
heart?!
!
A:!
Sinoatrial node!
B:!
Bundle of His!
C:!
Atrioventricular node!
D:!
Purkinje fibers!
You selected A; This is correct!!
!
Reason:!
!
Cardiac pacemakers are bundles of nerves that generate electrical impulses and conduct them
to the cardiac cells, resulting in contraction of the myocardium (heart muscle). In a normal
healthy heart, the sinoatrial (SA) node is the primary pacemaker that sets the inherent rate for
the heart. The SA node generates electricity at a rate of 60 to 100 electrical discharges per
minute, hence the normal adult heart rate of 60 to 100 beats/min. The atrioventricuar (AV) node
serves as the heart's secondary pacemaker; if the SA node fails, the AV node resumes the
pacing function of the heart, although at a slower rate (40 to 60 per minute). The bundle of His
and purkinjie fibers, located within the ventricles, may serve as tertiary pacemakers if the SA
and AV nodes fail; their inherent pacing rate is 20 to 40 per minute.!
!
=====================================!
!
The pain associated with acute aortic dissection:!
!
A:!
is typically described as a stabbing or tearing sensation.!
B:!
typically comes on gradually and progressively worsens.!
C:!
is usually preceded by nausea, sweating, and weakness.!
D:!
originates in the epigastrium and radiates down both legs.!
You selected A; This is correct!!
!
Reason:!
!
Aortic dissection occurs when the inner layers of the aorta become separated, allowing blood to
flow between the layers at high pressure. This separation of layers significantly weakens the
aortic wall, making it prone to rupture. Signs and symptoms of acute aortic dissection include a
sudden onset of a ripping, tearing, or stabbing pain in the anterior part of the chest or in the
back between the scapulae. It may be difficult to differentiate the pain of acute aortic dissection
from that of an acute myocardial infarction (AMI), but a number of distinctive features may help.
The pain from an AMI is often preceded by other signs and symptoms (ie, nausea, indigestion,
weakness, sweating [diaphoresis]). It tends to come on gradually and becomes more severe as
time progresses, and is usually described as a crushing pain or as a feeling of heaviness or
pressure. By contrast, the pain associated with aortic dissection is acute and is often of
maximum intensity from the onset; it is typically described as a ripping, tearing, or stabbing
sensation.!
!
=====================================!
!
Which of the following statements regarding one-rescuer CPR is correct?!
!
A:!
A compression to ventilation ratio of 15:2 should be delivered.!
B:!
The chest should be allowed to fully recoil after each compression.!
C:!
Ventilations should be delivered over a period of 2 to 3 seconds.!
D:!
You should assess the patient for a pulse after 3 cycles of CPR.!
You selected A; The correct answer is B;!
!
Reason:!
!
When performing CPR on any patient, you should allow the chest to fully recoil after each
compression. Incomplete chest recoil causes increased intrathoracic pressure, which may
impair blood return to the heart. Assess the patient's pulse after every 5 cycles (about 2
minutes) of CPR; take no longer than 5 to 10 seconds to do this. A compression to ventilation
ratio of 30:2 should be performed during all adult and one-rescuer CPR (adult, child, and infant),
except for newborns. A compression to ventilation ratio of 15:2 is used during two-rescuer infant
and child CPR. Ventilations should be delivered over a period of 1 second each, just enough to
produce visible chest rise.!
!
=====================================!
!
While transporting an elderly woman who was complaining of nausea, vomiting, and weakness,
she suddenly becomes unresponsive. You should:!
!
A:!
quickly look at her chest for obvious movement.!
B:!
analyze her heart rhythm with the AED.!
C:!
feel for a carotid pulse for at least 5 seconds.!
D:!
open her airway and ensure that it is clear.!
You selected A; This is correct!!
!
Reason:!
!
If a patient is found unresponsive or becomes unresponsive in your presence, your first action
should be to assess for breathing; this should be done by quickly (no more than 10 seconds)
looking at the chest for obvious movement. If the patient is not breathing or only has agonal
gasps, you should check for a pulse for at least 5 seconds but no more than 10 seconds. If the
patient has a pulse but is not breathing, open the airway and provide rescue breathing. If the
patient does not have a pulse, begin CPR (starting with chest compressions), and apply the
AED as soon as possible. If you are transporting a patient who becomes unresponsive,
pulseless, and apneic, you should begin CPR and instruct your partner to stop the ambulance
and prepare the AED.!
!
=====================================!
!
In patients with heart disease, acute coronary syndrome is MOST often the result of:!
!
A:!
coronary artery rupture.!
B:!
coronary artery spasm.!
C:!
atrial damage.!
D:!
atherosclerosis.!
You selected A; The correct answer is D;!
!
Reason:!
!
In most patients with acute coronary syndrome, or ACS (eg, unstable angina, acute myocardial
infarction), atherosclerosis is the underlying problem that causes heart disease. Atherosclerosis
is a disorder in which calcium and a fatty material called cholesterol build up and form a plaque
inside the walls of blood vessels, obstructing blood flow. ACS due to atherosclerosis usually
occurs when a fragment of plaque ruptures and occludes a coronary artery; further occlusion
occurs when platelets aggregate in the area and clump together. Less commonly, acute
coronary artery spasm may result in ACS. The cause of acute coronary vasospasm is largely
unknown. Rupture of a coronary artery is a rare cause of ACS. Atrial or ventricular damage is
usually caused by, rather than the cause of, ACS.!
!
=====================================!
!
After defibrillating a man in cardiac arrest, you resume CPR. As you are about to reanalyze his
cardiac rhythm 2 minutes later, your partner tells you she can definitely feel a strong carotid
pulse. You should:!
!
A:!
continue with the rhythm analysis.!
B:!
ask her to obtain a blood pressure reading.!
C:!
remove the AED pads from the patient’s chest.!
D:!
assess the patient’s breathing effort.!
You selected A; The correct answer is D;!
!
Reason:!
!
If return of spontaneous circulation (ROSC) occurs, your first action should be to reassess the
patient’s airway status and breathing effort. If the patient is still apneic or is breathing
inadequately, continue ventilations and frequently reassess his or her pulse. If the patient is
breathing adequately, apply high-flow oxygen via nonrebreathing mask. If the patient is still
unresponsive, as is often the case, insert an appropriate airway adjunct (if not already done) to
assist in maintaining airway patency. After reassessing airway and breathing, obtain the
patient’s blood pressure and treat hypotension if needed. Do not remove the AED pads from the
patient’s chest, even if ROSC has occurred. The risk of cardiac arrest is still high and the patient
may need further defibrillation. You should, however, disconnect the pads from the AED or
simply turn the AED off.!
!
=====================================!
!
You are caring for a 66-year-old woman with severe pressure in her chest. As you administer
oxygen to her, your partner should:!
!
A:!
notify medical control.!
B:!
gather her medications.!
C:!
obtain a set of vital signs.!
D:!
obtain a SAMPLE history.!
You selected A; The correct answer is C;!
!
Reason:!
!
After you have performed the primary assessment of a patient with a possible cardiac problem
and initiated treatment (ie, oxygen, aspirin), your partner should obtain a set of baseline vital
signs, which includes assessing the blood pressure, pulse, respiratory rate, and oxygen
saturation (SpO2). After the vital signs are obtained, the SAMPLE history is obtained, which
includes gathering the patient’s medications. Once you have completed your assessment,
including obtaining the vital signs and SAMPLE history, you should contact medical control if
guidance is needed.!
!
=====================================!
!
An elderly man in unresponsive and has agonal breathing. You should:!
!
A:!
perform rescue breathing at 10 to 12 breaths/min.!
B:!
check for a carotid pulse for up to 10 seconds.!
C:!
begin CPR, starting with chest compressions.!
D:!
open his airway and deliver 2 rescue breaths.!
You selected A; The correct answer is B;!
!
Reason:!
!
If a patient is unresponsive and is not breathing or has agonal breathing (also called agonal
gasps), you should check for a pulse for at least 5 seconds but no more than 10 seconds. If you
can definitely feel a pulse, provide rescue breathing (10 to 12 breaths/min for adults; 12 to 20
breaths/min for infants and children). If you cannot feel a pulse, or if you are unsure if a pulse is
present, begin CPR starting with chest compressions and apply the AED as soon as possible.!
!
=====================================!
!
A patient whose artificial pacemaker has failed would MOST likely experience:!
!
A:!
dizziness and excessive tachycardia!
B:!
irreversible ventricular fibrillation.!
C:!
hypertension and a headache.!
D:!
weakness and bradycardia.!
You selected A; The correct answer is D;!
!
Reason:!
!
An artificial pacemaker is implanted in a person whose own cardiac electrical conduction system
cannot maintain a regular rhythm and rate. If a pacemaker stops working, as when the battery
wears out or an internal lead becomes detached, the patient often experiences syncope,
dizziness, or weakness because of an excessively slow heart rate (bradycardia). The pulse rate
is typically less than 60 beats/min because the heart is beating without the stimulus of the
pacemaker and without regulation of its own electrical conduction system, which may be
damaged. In these circumstances, the heart tends to assume a fixed slow rate that may not be
fast enough to maintain adequate cardiac output. In some cases, the patient’s heart rate may be
so low that he or she becomes hypotensive.!
!
=====================================!
!
Ischemic heart disease is a condition in which:!
!
A:!
the coronary arteries dilate, thus preventing effective blood flow to the heart.!
B:!
an acute event leads to a significant decrease in the pumping force of the heart.!
C:!
a portion of the heart muscle dies because of a prolonged lack of oxygen.!
D:!
there is a decrease in blood flow to one or more portions of the heart muscle.!
You selected A; The correct answer is D;!
!
Reason:!
!
Chest pain or discomfort that is related to the heart usually stems from a condition called
ischemia (insufficient oxygen). Because of a partial or complete blockage of blood flow through
one or more coronary arteries, the tissue of the heart muscle (myocardium) fails to get enough
oxygen and nutrients relative to its needs. Therefore, ischemic heart disease is a condition
involving a decrease in blood flow, and therefore oxygen, to one or more portions of the
myocardium. If blood flow to the ischemic portion of the myocardium is not restored, it
eventually dies (myocardial infarction). Dilation of the coronary arteries increases, not
decreases, blood flow to the heart. If an event such as a myocardial infarction damages the
heart and significantly decreases its ability to contract forcefully, heart failure may occur.!
!
=====================================!
!
Prior to administering nitroglycerin to a patient with chest pain, you should:!
!
A:!
inquire about an allergy to salicylates.!
B:!
elevate the patient's lower extremities.!
C:!
auscultate the patient's breath sounds.!
D:!
obtain vital signs to detect hypotension.!
You selected A; The correct answer is D;!
!
Reason:!
!
Prior to assisting a patient with his or her prescribed nitroglycerin, there are two things that you
must do: take the patient's vital signs and obtain authorization from medical control.
Nitroglycerin is contraindicated for patients with a systolic blood pressure that is less than 100
mm Hg. If the patient develops hypotension after being given nitroglycerin, elevating his or her
lower extremities would be indicated. Salicylates are a class of drugs that include aspirin, not
nitroglycerin (nitroglycerin is a nitrate). Although you should inquire about medication allergies in
general, it is not necessary to inquire specifically about an allergy to salicylates unless you are
going to administer aspirin. Assessment of a patient with a possible cardiac or respiratory
problem should include auscultation of breath sounds; however, this does not necessarily have
to be done before assisting the patient with his or her nitroglycerin. !
!
=====================================!
!
A 66-year-old female presents with an acute onset of confusion, slurred speech, and weakness
to her right arm and leg. Her airway is patent and she is breathing adequately. The MOST
important initial information to determine regarding this patient is:!
!
A:!
her initial blood pressure reading.!
B:!
whether or not her pupils are equal.!
C:!
when her symptoms were first noted.!
D:!
what she was doing when this began.!
You selected A; The correct answer is C;!
!
Reason:!
!
This patient is experiencing signs of an acute ischemic stroke. She may be a candidate for
fibrinolytic therapy, drugs that destroy the clot in the cerebral artery, if her symptoms are of less
than 3 hours’ duration. It is vital to determine exactly (or as close to as possible) when the
patient’s symptoms were first noted, and pass this information along to the receiving facility.
Few, if any, current treatments are effective if they are started more than 3 to 6 hours after the
stroke begins. Even if 3 hours have passed, prompt action on your part is essential. Assessment
of the patient’s blood pressure, pupils, and events that preceded the symptoms are important;
however, identifying the patient as a candidate for an intervention that may reverse the stroke is
critical and will afford her the greatest chance for a positive outcome.!
!
=====================================!
!
A 56-year-old man is found to be pulseless and apneic. His wife states that he collapsed about 5
minutes ago. As your partner gets the AED from the ambulance, you should:!
!
A:!
open the airway and give 2 rescue breaths.!
B:!
provide rescue breaths until the AED is ready.!
C:!
ask the wife if the patient has a living will.!
D:!
begin CPR, starting with chest compressions.!
You selected A; The correct answer is D;!
!
Reason:!
!
When you arrive on scene and determine that a patient is in cardiac arrest, you should
immediately begin CPR, starting with chest compressions. Perform 30 chest compressions and
then open the airway and deliver 2 rescue breaths. Chest compressions are a crucial part of
cardiopulmonary resuscitation and must be started without delay. Apply the AED as soon as it is
available. In the interest of this patient, whose arrest interval is short, you should begin
resuscitative efforts immediately. In some cases, it is appropriate to inquire about the presence
of a living will; however, this should be done after resuscitative efforts have begun.!
!
=====================================!
!
A middle-aged woman took three of her prescribed nitroglycerin tablets after she began
experiencing chest pain. She complains of a bad headache and is still experiencing chest pain.
You should assume that:!
!
A:!
her nitroglycerin is no longer potent.!
B:!
she has ongoing cardiac ischemia.!
C:!
her chest pain is not cardiac-related.!
D:!
her blood pressure is elevated.!
You selected A; The correct answer is B;!
!
Reason:!
!
A headache and/or a bitter taste under the tongue are common side effects of nitroglycerin
(NTG) that many patients experience. If the patient does not experience these side effects, the
NTG may have lost its potency. However, if a patient with chest pain takes NTG and
experiences these side effects, but still has chest pain, you should assume that his or her pain
is the result of cardiac ischemia, a relative deprivation of oxygen to the heart. NTG is a
vasodilator drug; if anything, three doses would lower her blood pressure, not raise it. Any
patient with nontraumatic chest pain or pressure should be assumed to be experiencing cardiac
ischemia, especially if the pain or pressure is not relieved with NTG.!
!
=====================================!
!
When performing two-rescuer CPR on an adult patient whose airway has not been secured with
an advanced device, you should:!
!
A:!
deliver ventilations at a rate of 8 to 10 breaths/min.!
B:!
continue ventilations as the AED analyzes the patient’s cardiac rhythm.!
C:!
have your partner pause after 30 compressions as you give 2 breaths.!
D:!
avoid synchronizing compressions with ventilations!
You selected A; The correct answer is C;!
!
Reason:!
!
When performing two-rescuer adult CPR, you should perform cycles of CPR, with a
compression to ventilation ratio of 30:2. If the airway is not secured with an advanced device
(eg, ET tube, multilumen airway, supraglottic airway) ventilations and chest compressions
should be coordinated (synchronous). After your partner delivers 30 compressions, he or she
should pause as you deliver two breaths. After the airway has been secured with an advanced
device, do not attempt to synchronize compressions and ventilations. Compressions should be
performed continuously at a rate of at least 100/min and ventilations should be given at a rate of
8 to 10 breaths/min (one breath every 6 to 8 seconds). All contact with the patient must cease
as the AED is analyzing the cardiac rhythm.!
!
=====================================!
!
A 60-year-old man is in cardiac arrest. You begin CPR while your partner applies the AED. What
should you do if you receive a no shock message?!
!
A:!
Ensure that the AED electrodes are properly applied.!
B:!
Assess for a carotid pulse for up to 10 seconds.!
C:!
Reanalyze his cardiac rhythm after 30 seconds of CPR.!
D:!
Resume CPR, starting with chest compressions.!
You selected A; The correct answer is D;!
!
Reason:!
!
If the AED gives a no shock advised message, you should immediately resume CPR, starting
with chest compressions. After 2 minutes of CPR, reanalyze the patient’s cardiac rhythm and
follow the AED voice prompts. You should not assess for a pulse if the AED gives a no shock
message; this will only cause an unnecessary delay in performing chest compressions. Rarely,
if ever, does CPR alone restore a normal cardiac rhythm and pulse. If the AED electrodes are
improperly applied, it will not analyze the patient’s cardiac rhythm; instead, you will receive a
"check patient" or "check electrodes" message. Continue CPR, rhythm analysis every 2
minutes, and defibrillation (if indicated) until ALS personnel arrive or the patient starts to move.!
!
=====================================!
!
A 65-year-old man has generalized weakness and chest pressure. He has a bottle of prescribed
nitroglycerin, but states that he has not taken any of his medication. As your partner prepares to
administer oxygen, you should:!
!
A:!
perform a secondary assessment and obtain baseline vital signs.!
B:!
assist the patient with his nitroglycerin with medical control approval.!
C:!
apply the AED and prepare the patient for immediate transport.!
D:!
administer up to 325 mg of aspirin if the patient is not allergic to it.!
You selected A; The correct answer is D;!
!
Reason:!
!
Aspirin has clearly been shown to reduce mortality and morbitiy associated with acute coronary
syndrome (ACS) and should be given as early as possible; the dose is 160 to 325 mg. Even
though this patient has chest pain and prescribed nitroglycerin, you must first complete a
secondary assessment and obtain baseline vital signs. Medical control will need this information
—specifically the patient's blood pressure—in order to determine whether you should assist the
patient with his nitroglycerin. The AED is not indicated for this patient because he is not in
cardiac arrest.!
!
=====================================!
!
During your assessment of a 70-year-old man with crushing chest pain, you note that his blood
pressure is 80/50 mm Hg. Your MOST important action should be to:!
!
A:!
give high-flow oxygen.!
B:!
keep the patient warm.!
C:!
assess his oxygen saturation.!
D:!
transport without delay.!
You selected A; The correct answer is D;!
!
Reason:!
!
Patients with chest pain, pressure, or discomfort with a systolic BP less than 100 mm Hg should
be transported to the hospital without delay. Hypotension in a patient with chest pain indicates
cardiogenic shock due to severe cardiac damage and requires treatment that can only be given
at the hospital. Any delay in transport delays definitive care and increases the patient’s chance
of death. High-flow oxygen, thermal management, and assessment of oxygen saturation are
clearly important; however, your primary focus should be to get the patient to the hospital as
soon as possible.!
!
=====================================!
!
Which of the following is the BEST indicator of cardiac output?!
!
A:!
Pulse rate and quality!
B:!
Condition and color of the skin!
C:!
Quality of the respirations!
D:!
Systolic blood pressure!
You selected A; This is correct!!
!
Reason:!
!
Cardiac output is the amount of blood ejected from the ventricles each minute. To best obtain an
indication of cardiac output, you should assess the rate and quality of the pulse. A rapid,
bounding pulse indicates increased cardiac output, whereas a weak or “thready” pulse indicates
a decreased cardiac output. Changes in pulse quality (strength) will likely be detected before a
drop in systolic blood pressure.!
!
=====================================!
!
Your partner has applied the AED to a cardiac arrest patient and has received a shock advised
message. While the AED is charging, you should:!
!
A:!
cease all contact with the patient until the AED has delivered the shock.!
B:!
retrieve the airway equipment and prepare to ventilate the patient.!
C:!
continue chest compressions until your partner tells you to stand clear.!
D:!
perform rescue breathing only until the AED is charged and ready to shock.!
You selected A; The correct answer is C;!
!
Reason:!
!
It is important to minimize interruptions in CPR, especially chest compressions, when at all
possible. All contact with the patient must cease while the AED is analyzing. However, if the
AED gives a shock advised message and begins charging, you should resume chest
compressions until the AED is charged and ready to deliver the shock; at this point, you should
cease contact with the patient. As soon as the AED delivers the shock, resume CPR starting
with chest compressions.!
!
=====================================!
!
You arrive at the scene of a 56-year-old man who collapsed. The patient's wife tells you that he
suddenly grabbed his chest and then passed out. Your assessment reveals that he is apneic
and pulseless. As your partner begins one-rescuer CPR, you should:!
!
A:!
obtain a SAMPLE history.!
B:!
notify medical control.!
C:!
prepare the AED for use.!
D:!
insert an airway adjunct.!
You selected A; The correct answer is C;!
!
Reason:!
!
Immediate treatment for a patient in cardiac arrest involves performing CPR and applying the
AED as soon as possible. After applying the AED pads to the patient’s chest (around your
partner’s compressing hands), analyze his cardiac rhythm, deliver a shock if indicated, and
immediately resume CPR (starting with chest compressions). Management of the airway,
including insertion of an airway adjunct, should occur during the 2-minute period of CPR in
between cardiac rhythm analysis and defibrillation. While CPR is in progress, obtain as much of
the patient’s medical history from his wife as possible, and notify medical control when it is
practical (ie, you have more help at the scene).!
!
=====================================!
!
In which of the following patients is nitroglycerin contraindicated?!
!
A:!
53-year-old male with chest discomfort, diaphoresis, a blood pressure of 146/66 mm
Hg, and regular use of Levitra!
B:!
66-year-old female with chest pressure of 6 hours’ duration, lightheadedness, and a
blood pressure of 110/58 mm Hg!
C:!
58-year-old male with chest pain radiating to the left arm, a blood pressure of 130/64
mm Hg, and prescribed Tegretol!
D:!
41-year-old male with crushing substernal chest pressure, a blood pressure of 160/90
mm Hg, and severe nausea!
You selected A; This is correct!!
!
Reason:!
!
Nitroglycerin is contraindicated in patients who do not have a prescription for nitroglycerin, in
those with a systolic BP less than 100 mm Hg, and in patients who have taken medications for
erectile dysfunction (ED) within the previous 24 to 48 hours. Such medications include sildenafil
(Viagra), vardenafil (Levitra) and tadalafil (Cialis). Because ED drugs and nitroglycerin both
cause vasodilation, concomitant use of these drugs may result in significant hypotension.
Carbamazepine (Tegretol) is an anticonvulsant medication; there are no known interactions
between Tegretol and nitroglycerin.!
!
=====================================!
!
In addition to supplemental oxygen, one of the MOST effective way to minimize the detrimental
effects associated with acute coronary syndrome is to:!
!
A:!
request ALS support for any patient who has chest pain.!
B:!
administer nitroglycerin in 15 to 20 minute intervals.!
C:!
transport the patient rapidly, using lights and siren.!
D:!
reassure the patient and provide prompt transport.!
You selected A; The correct answer is D;!
!
Reason:!
!
In addition to increasing the body’s oxygen supply with supplemental oxygen, it is extremely
important to decrease oxygen demand and consumption. You can most effectively accomplish
this by keeping the patient calm, providing reassurance, and providing safe, prompt transport to
the hospital. Traveling at a high rate of speed with lights flashing and siren blasting would clearly
increase the patient’s anxiety and the heart’s demand for oxygen. The decision to request ALS
support is based on the patient’s condition and your transport distance to the closest
appropriate hospital. Unless contraindicated (ie, the patient is hypotensive, the medication is not
prescribed to the patient), nitroglycerin should be given in 5-minute intervals, up to three (3)
doses.!
!
=====================================!
!
A 45-year-old woman calls EMS because of severe chest pain. When you arrive, she advises
you that she has taken two of her husband’s nitroglycerin (NTG) tablets without relief. What is
your MOST appropriate course of action?!
!
A:!
Attach the AED, administer 100% oxygen, and contact medical control for advice.!
B:!
Apply oxygen, assess the patient's blood pressure, and give a third and final NTG tablet.!
C:!
Call medical control and request permission to assist the patient with one more NTG
tablet.!
D:!
Apply supplemental oxygen and transport the patient to the hospital without delay.!
You selected A; The correct answer is D;!
!
Reason:!
!
You should provide supplemental oxygen and prompt transport to any patient who reports chest
pain and does not have prescribed nitroglycerin. An EMT who knowingly administers someone
else’s medication to a patient could be held negligent. Medical control should always be
contacted when in doubt. However, bear in mind that medical control will not allow you to assist
a patient with someone else’s medication. The AED is only applied to patients who are in
cardiac arrest; the patient in this scenario is not in cardiac arrest.!
!
=====================================!
!
Which of the following describes pulseless electrical activity (PEA)?!
!
A:!
A rapid cardiac rhythm that does not produce a pulse, but responds to defibrillation!
B:!
A disorganized, chaotic quivering of the heart muscle that does not generate a pulse!
C:!
The presence of a palpable pulse in the absence of any electrical activity in the heart!
D:!
Any organized cardiac rhythm, slow or fast, that does not produce a palpable pulse!
You selected A; The correct answer is D;!
!
Reason:!
!
Pulseless electrical activity (PEA) is a condition in which the heart produces organized electrical
activity (slow or fast), despite the absence of a palpable pulse. A disorganized, chaotic quivering
of the heart muscle that does not generate a pulse is called ventricular fibrillation (V-Fib), and is
treated with defibrillation. Defibrillation is not indicated for patients with PEA; it is only indicated
for patients with V-Fib or pulseless ventricular tachycardia (V-Tach). If the AED gives a no shock
message, and the patient is still pulseless, he or she is either in asystole or PEA, neither of
which are shockable rhythms.!
!
=====================================!
!
Which of the following assessment findings is LEAST suggestive of cardiac compromise?!
!
A:!
Anxiety and pale, cool skin!
B:!
Rapid, irregular heart rate!
C:!
Nausea and epigastric pain!
D:!
Palpable pain to the chest!
You selected A; The correct answer is D;!
!
Reason:!
!
Signs and symptoms of cardiac compromise include nonreproducible pain, pressure, or
discomfort in the chest or epigastric region; nausea; pale, cool, clammy (diaphoretic) skin; and
an irregular pulse that is either fast or slow. Pain of cardiac origin typically is not reproducible by
palpation. Palpable pain to the chest suggests a musculoskeletal problem, not a cardiac
problem. However, because some patients with a cardiac problem present atypically, you should
transport any patient with chest pain, pressure, or discomfort to the hospital.!
!
=====================================